Twice the sum of two consecutive integers is 22. What are the two integers?

Answers

Answer 1
It would be 5 and 6, because consecutive integers follow each other, so twice the sum of 5 and 6 would be 22.

Related Questions

Complete the following, using ordinary interest. (Use Days in a year table.) (Do not round intermediate calculations. Round the "Interest" and "Maturity value" to the nearest cent.)
principal 1,000 interest rate 8% Date borrowed March 8 Date repaid June 9

Exact time? interest? Maturity value?

Answers

The exact time is 93 days

The interest is  $20.38

The maturity value is  $1020.38

What is the interest and the maturity value?

The interest is a function of the time, amount borrowed and the interest rate.

Interest = amount borrowed x time x interest rate

Time = June 9 - March 8 = 93 days

1000 x 0.08 x (93/365) = $20.38

Maturity value = 1000 + 20.38 = $1020.38

To learn more about interest, please check: https://brainly.com/question/26164549

#SPJ1

Urgent!!!
If the sum of 5 consecutive positive integers is w , what is the sum of the next 5 consecutive positive integers in terms of w?


A. w+25 B. w+23 C. w+27 D. w+29

Answers

The sum of the next 5 consecutive positive integers is w + 25

How to determine the sum of the next numbers?

Let the first five numbers be A - E

So, we have:

A + B + C + D + E = W

The next five numbers would be

A + 5, B + 5...... E + 5

So, we have:

Sum = A + 5 + B + 5 + C + 5 + D + 5 + E + 5

Evaluate the like terms

Sum = A + B + C + D + E + 25

Substitute A + B + C + D + E = W

Sum = W + 25

Hence, the sum of the next 5 consecutive positive integers is w + 25

Read more about consecutive integers at:

https://brainly.com/question/10853762

#SPJ1

Which value is equivalent to 48 ÷ ((2 + 6) × 2) − 1? Group of answer choices 2 -12 3 3 1/5

Answers

The solution to the given expression is 2

Solving equation

Given the equation 48 ÷ ((2 + 6) × 2) − 1

Using PEMDAS

48 ÷ ((8) × 2) − 1

48÷(16) - 1

3 - 1

2

Hence the solution to the given expression is 2

Learn more on equation here: https://brainly.com/question/13763238

#SPJ1

f(x)=5x/x-25 asymptotes and state the end behavior of the function

Answers

The asymptote of the function given in the task content is the line; x = 25.

What is the asymptote of the function?

It follows from the task content that the function given is a rational function and hence, the asymptote of the function is the X-value which renders the function undefined in which case, the denominator is equal to 0.

x - 25 = 0.

x = 25.

The end behaviour of the functions graph is that it approaches the line x = 25 but never touches the line.

Read more on asymptote;

https://brainly.com/question/4138300

#SPJ1

Which one of the following situations could be represented by a linear function?
A. Number of bacteria is doubled every day.
B. Jacob drives a car and covers the same distance for every hour.

Answers

the answer A for a linear function

Which expressions are equivalent to the one below? Check all that apply.
49x

Answers

The equivalent expressions are (b) (7 * 7)^x , (c) 7^x * 7^x and (e) 7^(2x)

How to determine the equivalent expressions?

The expression is given as:

49^x

Express 49 as 7 * 7

(7 * 7)^x ---- option (b)

Express 7 * 7 as an exponent

7^(2x) ---- option (e)

Express 2x as x + x

7^(x +x)

Expand

7^x * 7^x --- option (c)

Hence, the equivalent expressions are (b), (c) and (e)

Read more about equivalent expressions at:

https://brainly.com/question/27911936

#SPJ1

Aimee is shipping a care package to her friend in college, and she has a piece of
cardboard with which to create a box. A rectangular cardboard sheet measuring 20
inches by 40 inches is to be used by cutting out square corners of measurex inches.
40 inches
XI
X
L.
20 inches
I

Answers

Aimee is shipping a care package to her friend in college, the max volume of the box is mathematically given as

V=1539.6007

What is the max volume of the box?

Generally, the equation for the height is mathematically given as

V=(20-2x)(40-2x)x

Where x is the height

V=4x^3-120x^2+800x

differentiate v and solve

dv/dx=3x^2-60+200

3x^2-60+200=0

Therefore

x=[tex]\frac{60\pm \sqrt{60^2-4x3x200}}{2*3}[/tex]

x=[tex]\frac{60-34.64}{6}[/tex]

x=4.2265

In conclusion, the second differentiation of v and solve we have

d^2v/dx^2=24-240

Hence

d^2v/dx^2< 0

V=(20-2x(40-2x)x

subsitute x

V=(20-2(4.2265))(40-2(4.2265))(4.2265)

V=1539.6007

Read more about volume

https://brainly.com/question/13338592

#SPJ1

If a translation of T2, –7(x, y) is applied to ΔABC, what are the coordinates of B'?

(1, –5)
(–4, –10)
(–3, –12)
(3, –12)

Answers

The coordinates of B' are 3 and -12. so option D is correct (3, -12)

How to find the coordinates of B'?

The point P(x, y) may undergo translation to form P' by using a defined translation rule :

If given the rule T(a, b) synch that P undergoes translation on T; then we will have P'(x+a; y+b)

Therefore,

The point B has its coordinates as B(1, - 5)

Translation rule, T(2, - 7) then the coordination of B after undergoing transformation according to T2;

B(1, - 5) - - - > T(2, - 7) - - - - > B'(1+2 ; - 5+-7)

= B'(3, -12)

Hence, the coordinates of B' are 3 and -12. so option D is correct.

Learn more about the concept :

brainly.com/question/12463306

#SPJ1

Answer:

(D)   (3, –12)  

Step-by-step explanation:

If a translation of T2, –7(x, y) is applied to ΔABC, what are the coordinates of B'? The answer is (3, –12).

You need a 25% alcohol solution. On hand, you have a 225 mL of a 35% alcohol mixture. How much pure water will you need to add to obtain the desired solution?
You will need____ mL of pure water to obtain___ mL of the desired 25% solution.

Answers

You will need 252 mL of pure water to obtain 0.25 mL of the desired 25% solution.

If we add X amount of water to the solution, the total amount of solution is 375+X.

And that must be 25%.

What is the amount of solute in the current solution?

The amount of solute in the current amount of solution is

0.6(375)=225.

That amount won't change by adding water.

So we set up the equation so that the amount of solute divided by the new total amount of solution equals 25%.

225/(375+X)=0.25

cross multiply and solve

225/0.25=(375+X)

900-375=X

X=525

The pure water will you need to add to obtain the desired solution is 525.

Then we check 225/(375+525)=0.25.

To learn more about the mixture problem visit:

https://brainly.com/question/24647756

#SPJ1

which statement is true?
12/13 < 36/30
13/17>24/28

Answers

Answer:

The first one: 12/13  > 36/30

Step-by-step explanation:

The first one would be correct as 12/13 is less than 36/30.

To explain why 13/17 is actually less than 24/28, see below:

So when comparing fractions, we must follow steps.

Step One: convert mixed numbers to a fraction, if there are any

There are not mixed numbers to compare, so we can move to the next step.

Step Two: find the least common denominator of the two fractions

Least common denominator = 476

Check out our least common denominator calculator for help on finding this.

Step Three: rewrite each fraction to an equivalent fraction using the denominator 476

To do this, start by dividing 476 by the denominator of the first fraction. Next, multiply the result by the numerator to find the new numerator. To rewrite, put the new numerator over 476. Repeat this for the second fraction

364/476                              408/476

Step Four: compare the numerators

At this point, to compare the fractions, we can simply compare the numerators to see which is larger

364<408

Step Five: rewrite each fraction as the original fraction

13/17<24/28

Therefore, the first statement (12/13 > 36/30) is true.

Answer:

  12/13 < 36/30

Step-by-step explanation:

There are many ways to compare numbers. One way is to compare them to some value that lies between. Another is to consider their difference from a value that does not lie between.

12/13 vs 36/30

In the first fraction, 12 < 13, so the value of the fraction is less than 1:

  12/13 < 1

In the second fraction, 36 > 30, so the value of the fraction is greater than 1:

  1 < 36/30

Then the order of the fractions is ...

  12/13 < 1 < 36/30

  12/13 < 36/30 . . . . . the given order is True

13/17 vs 24/28

We notice the difference between numerator and denominator is 4 in each case, so we can write each fraction as a difference from 1:

  (13/17) vs (24/28)

  = (1 - 4/17) vs (1 -4/28) . . . . . fractions rewritten

  = -4/17 vs -4/28 . . . . . . . . . subtract 1 from both

The first fraction, 4/17, has a smaller-value denominator, so its magnitude is larger than that of the fraction 4/28. In other words, the ordering of these fractions is ...

  -4/17 < -4/28

  1 -4/17 < 1 -4/28 . . . . . . add 1 to both sides

  13/17 < 24/28 . . . . . the given order is False

__

Additional comment

One sort of "no brainer" way to compare the fractions is to multiply them by the product of their denominators. This looks like "cross multiplication" where each numerator is multiplied by the opposite denominator. As long as you keep the numerators in the same relative places, the comparison symbol will be the correct one for the fractions.

  12/13 vs 36/30   ⇒   (12·30) vs (13·36)   ⇒   360 < 468

The left fraction is smaller than the right fraction.

Similarly, ...

  13/17 vs 24/28   ⇒   (13·28) vs (17·24)   ⇒   364 < 408

The left fraction is smaller than the right fraction.

__

Here, we have used the fractions "as is." In each case, the fraction on the right could be reduced, possibly making the comparison easier.

What is the image of A(5,6) under R90°?

Answers

For the coordinate (5,6), the 90 degree rotation will be equivalent to (-6, 5)

Rotation of images

Rotation is a technique used to change the position of an object on an xy-plane.

If the coordinate point (x, y) is rotated 90degrees, the rule will be:

(x, y) -> (-y, x)

For the coordinate (5,6), the 90 degree rotation will be equivalent to (-6, 5)

Learn more on rotation here: https://brainly.com/question/26249005

#SPJ1

Which of the following options is an even function?

Answers

Using it's concept, an even function is given by:

B. [tex]F(x) = 2\cos{\frac{1}{2}x}[/tex]

What are even functions?

Even functions are functions for which the following statement is true for all values of x.

f(x) = f(-x).

The cosine function, without a phase shift, is an example of an even function, hence option B is correct.

More can be learned about even functions at https://brainly.com/question/3964641

#SPJ1

Determine the units digit (ones digit) of the counting number represented by the exponential expression.
4^800

Answers

The units digit of the given exponential expression will be 6

Determining the units digit of an exponential expression

From the question, we are to determine the units digit of the given exponential expression

The given exponential expression is

4⁸⁰⁰

We know that

4¹ = 4

4² = 16

4³ = 64

4⁴ = 256

4⁵ = 1024

4⁶ = 4096

From above, we can observe that the units digit of the odd exponents is 4 while the units digit of the even exponents is 6.

The exponent of the given expression is 800. 800 is an even number.

Hence, the units digit of the given exponential expression will be 6

Learn more on Determining units digits here: https://brainly.com/question/15419249

#SPJ1

look at the picture

Answers

The end behaviour of the function is When x ---> ∞ , f(x) ---> -∞  , when x ---> -∞ f(x) ----> ∞  , Option D is the correct answer.

What is a Function ?

A function is a mathematical statement that related the independent and the dependent variable.

The function given is

f(x) = [tex]\rm -2 \sqrt[3]{x}[/tex]

To understand the end behaviour the graph is plotted

When x ---> ∞ , f(x) ---> -∞

when x ---> -∞ f(x) ----> ∞

Therefore Option D is the correct answer.

To know more about Function

https://brainly.com/question/12431044

#SPJ1

The factor tree below provides the factors of 18. A factor tree of 18. 18 branches to 2 and 9. 9 branches to 3 and 3.

Answers

The prime factorization of 18 is 2 x 3 x 3.

What is the prime factorization?

A prime number is a number that is only divisible by 1 and itself. An example is 11. The factors of a number are numbers that divide another number without leaving any remainder. For example, a factor of 10 is 2. The prime factor of a number is the factors of a number that are prime numbers.

The factors of 18 = 1, 2, 3, 6, 9, 18

Prime factors = 1, 2, 3

Prime factorisation = 2 x 2 x 3

Here is the complete question:
The factor tree below provides the factors of 18. A factor tree of 18. 18 branches to 2 and 9. 9 branches to 3 and 3. What is the prime factorization of 18? 2 times 3 2 times 9 2 times 3 times 3 2 times 3 times 9

To learn more about factors, please check: https://brainly.com/question/26073850

#SPJ1

HELP
A small radio transmitter broadcasts in a 50 mile radius. If you drive along a straight line from a city 56 miles north of the transmitter to a second city 58 miles east of the transmitter, during how much of the drive will you pick up a signal from the transmitter?

Answers

Given the radius of 50 miles and the line joining the cities at (0, 56) and (58, 0), the transmitter signal can be picked during 59.24 miles of the drive.

How can the duration of signal reception be found?

Radius of broadcast of the transmitter = 50 miles

Location of starting point = 56 miles north of the transmitter

Location of destination city = 58 miles east of the transmitter

Therefore we have;

Slope of the line joining the two cities

= 56 ÷ (-58) = -0.966

Which gives the equation of the line as follows;

y = -0.966•x + 56

The equation of the circle is;

[tex] {x}^{2} + {y}^{2} = {50}^{2} [/tex]

[tex] {x}^{2} + { ( - 0.966x + 56)}^{2} = {50}^{2} [/tex]

1.933156•x^2 - 108.192•x + 636 = 0

Which gives;

x = 6.67 or x = 49.29

Therefore;

When x = 6.67, we have;

y = -0.966 × 6.67 + 56 = 49.56

When x = 49.29, we have;

y = -0.966 × 49.29 + 56 = 8.4

The length of the drive, during which the driver can pick the signal, l, is therefore;

l = √((49.56-8.4)^2 + (49.29-6.67)^2) = 59.24 miles

The length of the drive during which the signal is received is 59.24 miles

Learn more about the equation of a circle here:

https://brainly.com/question/502872

#SPJ1

Marina paid $16.95 for a sweater on sale which is 65% of the original cost. How much did the sweater cost originally

Answers

Answer:

$26.08.

Step-by-step explanation:

[tex]1. \ \frac{16.95}{0.65} =\frac{x}{1};[/tex]

2) x=16.95/0.65≈$26.076923076923076923

Solve the inequalities by graphing. Select the correct graph. y > 2x y < 3

Answers

The solution of the first graph is 1, 2 while the second graph is a straight line.

How to solve for the graph

We have y > 2x

y< x

First you have to replace the inequality by a mathematical operator

y - 2x = 0

y - 3 = 0

equation 1 is of the form

y = mx

The slope here is 2,

to make a graph we need to have x = 1 then the line would pass through

(1,2).

The second equation is simply a second line that would pass through the x axis.

The correct graph is below:

Read more on graphs here:

https://brainly.com/question/14543557

#SPJ1

Graph the image using the given transformation

Answers

The triangle KLM is rotated about origin by 90 degrees to form the triangle K'L'M'.

What is a transformation of geometry?

A spatial transformation is each mapping of feature shapes to itself, and it maintains some spatial correlation between figures.

Rotation does not change the size and shape of the geometry.

The triangle KLM is rotated about origin by 90 degrees to create the triangle K'L'M'.

The diagram is given below.

More about the transformation of geometry link is given below.

https://brainly.com/question/22532832

#SPJ1

The diameter of a proton is about 1.9 cross times 10 to the power of short dash 15 end exponent meters. A hydrogen atom has an overall length of 100,000 times (or 1 cross times 10 to the power of 5 times) the diameter of a proton.
What is the length of the hydrogen atom, in meters, if it were written in scientific notation?

A. 1.9 x 10 (15 exponent)
B. 1.9 x 10 (-15 exponent)
C. 1.9 x 10 (-8 exponent)
D. 1.9 x 10 (-12 exponent)

Answers

Using scientific notation, the length of the hydrogen atom is given as follows:

[tex]1.9 \times 10^{-10}[/tex]

What is scientific notation?

A number in scientific notation is given by:

[tex]a \times 10^b[/tex]

With the base being [tex]a \in [1, 10)[/tex].

The diameter of a proton is given by:

[tex]1.9 \times 10^{-15}[/tex].

The hydrogen atom has an overall length of 100,000 times the diameter of the proton, hence the length is given by:

[tex]L = 100000 \times 1.9 \times 10^{-15} = 10^5 \times 1.9 \times 10^{-15} = 1.9 \times 10^{5 - 15} = 1.9 \times 10^{-10}[/tex]

More can be learned about scientific notation at https://brainly.com/question/16394306

#SPJ1

A textbook store sold a combined total of 460 physics and math textbooks in a week. The number of math textbooks sold was 62 less than the number of physics textbooks sold. How many textbooks of each type were sold?

Answers

Number of physics textbooks sold was 261 & number of maths textbooks sold was 199.

Let, the number of physics textbooks sold = x

According to the question,

The number of math textbooks sold was 62 less than the number of physics textbooks sold.

So, Number of maths textbooks sold = x-62

The textbook store sold a combined total of 460 physics and math textbooks in a week.

So, the equation becomes,

(x-62)+x = 460

⇒ x-62+x = 460

⇒ 2x-62 = 460

⇒ 2x = 460+62

⇒ 2x = 522

⇒ x = 522/2

⇒ x = 261

So, 261 physics textbooks were sold

(261-62) = 199 maths textbooks were sold.

Learn more about equation here :

https://brainly.com/question/27893282

#SPJ10

What is the numerator of the simplified sum?
X
x²+3x+2x+1
3
O x+3
O 3x+6
O 4x+6
O 4x+2

Answers

Answer:

4x+6

Step-by-step explanation:

x/x^2+3x+2 + 3/x+1

x/(x+1)(x+2) + 3/x+1

make the denominator the same

x/(x+1)(x+2) + 3 (x+2)/(x+1)(x+2)

x/(x+1)(x+2) + 3x+6/(x+1)(x+2)

4x+6/(x+1)(x+2)

https://brainly.com/question/4160054

The requried, numerator of the simplified sum is 4x + 6. The option is correct.

What is simplification?

Simplification involves applying rules of arithmetic and algebra to remove unnecessary terms, factors, or operations from an expression. The goal is to obtain an expression that is easier to work with, manipulate, or solve. For example, simplifying an algebraic expression might involve combining like terms, factoring, or using the distributive property

The given expression is x/(x²+3x+2) + 3/(x+1). To make the denominators the same, we need to factor the denominator of the first fraction:

x/(x+1)(x+2) + 3/(x+1)

Now we can add the two fractions by finding a common denominator, which is (x+1)(x+2):

x/(x+1)(x+2) + 3(x+2)/(x+1)(x+2)

Simplifying the numerators, we get:

x + 3x + 6 / (x+1)(x+2)

Combining like terms in the numerator, we get:

4x + 6 / (x+1)(x+2)

Therefore, the simplified expression is (4x+6)/(x+1)(x+2).

Learn more about simplification here:

https://brainly.com/question/12501526

#SPJ5

What is the value of the expression shown 7 + (2 + 6)^2 divided by 4 x (1/2)^4

Answers

The value of the expression is 188

How to determine the value

Given the expression

= [tex]\frac{7 + (2 + 6)^2}{4 * \frac{1}{2}^4}[/tex]

Expand the bracket

= [tex]\frac{7 + 4 + 36}{ 4 * 1/16}[/tex]

We have

= [tex]\frac{47}{1/4}[/tex]

The denominator becomes the multiplier, we have

= 47 × 4

= 188

Thus, the value of the expression is 188

Learn more about algebraic expression here:

https://brainly.com/question/723406

#SPJ1

Find the area of the shaded region.
The outside of the figure is a rectangle that is 24 inches long and 8 inches high. The entire rectangle is shaded except for a triangle with a base of 24 inches and a height of 8 inches.

Answers

Answer:

96 sq. inch

Step-by-step explanation:

rectangle has 24 inches × 8 inches dimensions

Hence, Area of Rectangle,

A1=24×8

=192 sq. inch

We, know triangle with base 24 inches and height 8 inches has area exactly half of the rectangle's.

ie. A2=½(24×8)

=96 sq. inch

This was for unshaded region. Then remaining half area is for shaded region.

A1-A2=A

ie. 192-96=96 sq. inch

The area of the shaded region is 96 square inches.

How to determine the area of the shaded region

To find the area of the shaded region, we need to subtract the area of the triangle from the area of the rectangle.

The area of a rectangle is calculated by multiplying its length by its width. In this case, the rectangle has a length of 24 inches and a width of 8 inches, so its area is 24 inches * 8 inches = 192 square inches.

The area of a triangle is calculated by multiplying its base by its height and dividing the result by 2. In this case, the triangle has a base of 24 inches and a height of 8 inches, so its area is (24 inches * 8 inches) / 2 = 96 square inches.

Now, to find the area of the shaded region, we subtract the area of the triangle from the area of the rectangle:

Shaded Area = Area of Rectangle - Area of Triangle

= 192 square inches - 96 square inches

= 96 square inches

Therefore, the area of the shaded region is 96 square inches.

..

Learn more about area at https://brainly.com/question/25292087

#SPJ2

Hi, can someone please help me? trying to graduate :(
Write an equation in slope-intercept form of the line that passes through the point (-6,-5) with slope 6

Answers

Slope intercept form is y=mx+b, therefore your answer is D.

Hope this helped :)

The equation of the line in slope-intercept form that passes through the point (-6, -5) with a slope of 6 is y = 6x + 31. The correct answer is option (D).

Given that the line passes through the point (-6, -5) with a slope of 6, we can plug these values into the equation to find the y-intercept (b).

Using the point-slope form of the equation:

y - y₁ = m(x - x₁)

Substitute (-6, -5) and the given slope (m = 6):

y - (-5) = 6(x - (-6))

y + 5 = 6(x + 6)

Now, let's simplify the equation:

y + 5 = 6x + 36

To get the equation in slope-intercept form (y = mx + b), isolate y:

y = 6x + 36 - 5

y = 6x + 31

Thus, the correct answer is option (D).

Learn more about the Linear equations here:

https://brainly.com/question/13738061

#SPJ3

Your classmate tells you the details of the great deal he got on his mortgage: 30 year 3.5% fixed rate with 10% down payment how much is he going to borrow to buy the house (assuming he only has the money to make the down payment from the previous question)

Answers

Assuming he has the money to make the down payment, my classmate will borrow 90% of the home price.

What is a down payment?

A down payment is the upfront payment or an initial portion of the total purchase price of expensive property.

It is required that the buyer of a mortgaged property deposits this initial portion before taking the mortgage for the balance.

Data and Calculations:

Mortgage period = 30 years

Interest rate = 3.5% fixed

Down payment = 10%

Balance (Mortgage) = 90% (1 - 10%)

Thus, assuming he has the money to make the down payment, my classmate will borrow 90% of the home price.

Learn more about down payments at https://brainly.com/question/1698287

#SPJ1

If we run the following line:
y = int(3 * '4')

what is the value of y?

Answers

The value of y in the code segment is 444

How to determine the value of y?

The code segment is given as:

y = int(3 * '4')

Evaluate the expression in the bracket

y =int('444')

Remove the bracket

y = 444

Hence, the value of y in the code segment is 444

Read more about code segments at:

https://brainly.com/question/26683418

#SPJ1

Which number line represents the solution set for the inequality -1/2x>4?

Answers

Answer:

2nd Option

Step-by-step explanation:

Hello!

First, let's solve the inequality for x.

Solve for x[tex]-\frac12x \ge 4[/tex][tex]x \le-8[/tex]

Since the symbol is less than or equal to, it will be a closed circle on Point -8 going in the left direction (all values less than -8).

The answer is the Second Number Line.

A student guesses randomly on a 15​​-question multiple-choice quiz, where each question has 6​​ options. What is the probability that the student answers all questions correctly?

Answers

The probability that the students gets all the questions correctly is (1/6)^15 .

What is the probability?

Probability determines the odds that a random event would happen. The probability the event occurs is 1 and the probability that the event does not occur is 0.

The probability = (number of correct options/ total number of options)^number of questions

(1/6)^15

To learn more about probability, please check: https://brainly.com/question/13234031

#SPJ1

The integer having absolute value 10 are_________?

Answers

the answer is 10 and -10
Other Questions
Match the correct sentence together.Questions:10 pounds 20 feet =37 pounds 10 feet =29 newtons 10 meters =30 newtons 2 meters =Answers: 200 foot-pounds60 newton-meters290 newton-meters20 foot-pounds370 foot-pounds I need help pls WILL GIVE BRAINLIEST The sum of two numbers is 79. Three times the first number added to 5 times the second number is 283. What are the two numbers? A balanced dime is tossed three times. The possible outcomes are represented in the table. Complete parts (a) through (d) below.HHHHTHTHHTTHHHTHTTTHTTTTa. Find the probability that all three of the tosses come up heads. In what way could you increase the rate of the reaction as it taking place in image C?add more enzymeadd more substrateremove substrateremove enzymesremove product Question 3 of 25Which group of people was forced to leave their ancestral lands and move toreservations?A. Indentured servantsB. Native AmericansC. ImmigrantsD. Freed slaves Hippocrates, an ancient greek philosopher, is often referred to as the ______________ of medicine. True or false: GAAP require end-of-period adjustments for the estimated bad debts in the period of the credit sale even though the specific, non-paying customers have not yet been identified. True false question. True False Which was the largest culture-group in the Soviet Union? *ByelorussiansUkrainiansLithuaniansO Uzbeks The HPV vaccine is nearly 100% effective in preventing ___ strains of HPV that cause warts and cervical cancer My Activities Tell me what you want to do QWhich of the following scenarios does the free body diagram belowillustrate?Tension TensionWeightOA sign board supported by two strings.O A rubber band stretched by a person.O A door pulled by two applied forces.O A book falling off a table. Hemolysis due to poor venipuncture technique can cause falsely What is the approximate total length of iron edging needed to create the square frame and the two diagonals? 43.5 inches 50.9 inches 54.0 inches 61.5 inches Jamie is writing a research paper about wind patterns which digital resource should he consult is 7.666 a rational number Money demand depends on the price level and the interest rate. the price level but not the interest rate. the interest rate but not the price level. neither the price level nor the interest rate. In the United States approximately ______ percent of births occur in hospitals, with sterile procedures, electronic monitoring, and drugs to dull pain. The rationalist finally will be of dogmatic [rigid/arrogant] temper in his affirmations, while the empiricist may be more __________________ discussion A test requires that you answer first Part A and then either Part B or Part C. Part A consists of 6 true-false questions, Part B consists of 6 multiple-choice questions with one correct answer out of five, and Part C consists of 3 multiple-choice questions with one correct answer out of six. How many different completed answer sheets are possible? Write a proof given: angle 1 and angle 3 are supplementaryprove: m is parallel to n